Consider that we want to design a hash function for a type of message made of a sequence of integers like this M=(a 1

,a 2

,…,a t

). The proposed hash function is this: h(M)=(Σ i=1
t

a i

)modn where 0≤a i


(M)=(Σ i=1
t

a i
2

)modn c) Calculate the hash function of part (b) for M=(189,632,900,722,349) and n=989.

Answers

Answer 1

For the message M=(189,632,900,722,349) and n=989, the hash function gives h(M)=824 (based on the sum) and h(M)=842 (based on the sum of squares).

To calculate the hash function for the given message M=(189,632,900,722,349) using the formula h(M)=(Σ i=1 to t a i )mod n, we first find the sum of the integers in M, which is 189 + 632 + 900 + 722 + 349 = 2792. Then we take this sum modulo n, where n=989. Therefore, h(M) = 2792 mod 989 = 824.

For the second part of the hash function, h(M)=(Σ i=1 to t a i 2)mod n, we square each element in M and find their sum: (189^2 + 632^2 + 900^2 + 722^2 + 349^2) = 1067162001. Taking this sum modulo n=989, we get h(M) = 1067162001 mod 989 = 842.So, for the given message M=(189,632,900,722,349) and n=989, the hash function h(M) is 824 (based on the sum) and 842 (based on the sum of squares).



Therefore, For the message M=(189,632,900,722,349) and n=989, the hash function gives h(M)=824 (based on the sum) and h(M)=842 (based on the sum of squares).

To learn more about integers click here

brainly.com/question/18365251

#SPJ11


Related Questions

S1. First find all constant solutions, then find an explicit solution to the separable ODE:
y' (t) = y^2t^2
What is the specific solution that satisfies the initial condition y(0) = 3? What is the specific solution that satisfies the initial condition y(0) = 0 ?

Answers

The specific solution that satisfies the initial condition y(0) = 0 is:y(t) = -1 / 3t^3. The solution satisfies the initial condition y(0) = 0

We can start solving the separable differential equation, y'(t) = y^2t^2 as follows:

Separate the variables:

dy/y² = t²dtIntegrate both sides:

∫(dy/y²) = ∫t²dtWe get:

y^(-1) / -1 = t^3 / 3 + C1C1 is a constant of integration.

Rearrange to solve for y:y(t) = -1 / (3t^3 + 3C1)By applying the initial conditions:

y(0) = 3We can find a value for C1:

3 = -1 / (3*0^3 + 3C1)C1 = -1

Therefore, the specific solution that satisfies the initial condition y(0) = 3 is:

y(t) = -1 / (3t^3 - 3)Similarly, we can apply the second initial condition:

y(0) = 0We can find a value for C1:0 = -1 / (3*0^3 + 3C1)C1 = 0

To know more about specific solution visit:-

https://brainly.com/question/28649460

#SPJ11

Solve the equation. Check your solutions. (Enter your answers as a comma-separated list.) x^{6} −64=0
x =

Answers

Both solutions satisfy the equation, confirming their validity.

To solve the equation \(x^6 - 64 = 0\), we can factor it as a difference of squares:

\((x^3)^2 - 8^2 = 0\)

Now we have a difference of squares:

\((x^3 - 8)(x^3 + 8) = 0\)

Applying the difference of cubes formula, we can factor further:

\((x - 2)(x^2 + 2x + 4)(x + 2)(x^2 - 2x + 4) = 0\)

Setting each factor to zero, we find the following solutions:

\(x - 2 = 0\)   -->   \(x = 2\)

\(x^2 + 2x + 4 = 0\)   -->   This quadratic equation does not have real solutions.

\(x + 2 = 0\)   -->   \(x = -2\)

\(x^2 - 2x + 4 = 0\)   -->   This quadratic equation does not have real solutions.

Therefore, the solutions to the equation \(x^6 - 64 = 0\) are \(x = 2\) and \(x = -2\).

To check the solutions, we can substitute them back into the original equation:

For \(x = 2\):

\(2^6 - 64 = 64 - 64 = 0\)

For \(x = -2\):

\((-2)^6 - 64 = 64 - 64 = 0\)

Learn more about equation here:-

https://brainly.com/question/31251286

#SPJ11

The displacement (in centimeters) of a particie s moving back and forth along a straight line is given by the equation s=5 sin( xt ) +4 cos( πt ), where t is measured in seconds. (Round your answers to two decimal places.) (a) Find the average velocity during each time period. (i) [1,2] cm/s (ii) [1,1.1] x cm/s (ii) [1,1,01] x em/s. (iv) [1,1,001] x cmvs (b) Estimate the instantancous velocty of the particle when t=1. X cmis

Answers

The displacement (in centimeters) of a particle s moving back and forth along a straight line is given by the equation [tex]s=5 sin( xt ) +4 cos( πt )[/tex],

where t is measured in seconds. Therefore, the instantaneous velocity of the particle when t = 1 is approximately 2.35x cm/s.

To find the average velocity during each time period follow the steps given below:Given equation of displacement of the particle,

[tex]s(t) = 5sin(xt) + 4cos(πt)[/tex]

[tex]vavg = [s(2) - s(1)]/(2 - 1)[/tex]

= s(2) - s(1)

= [tex][5sin(2x) + 4cos(πx)] - [5sin(x) + 4cos(π)][/tex]

= [tex]5sin(2) - 5sin(1) + 4(cos(π) - cos(π))[/tex]

=[tex]5(sin(2) - sin(1)) cm/s≈ 0.61 cm/s[/tex]

(ii) The average velocity during time period [1,1.1] is given by;

[tex]vavg = [s(1.1) - s(1)]/(1.1 - 1)[/tex]

= s(1.1) - s(1)

= [tex][5sin(1.1x) + 4cos(π1.1)] - [5sin(x) + 4cos(π)][/tex]

= [tex]5sin(1.1) - 5sin(1) + 4(cos(π1.1) - cos(π))[/tex]

= 5(sin(1.1) - sin(1)) cm/s≈ 0.44 cm/s

(iv) The average velocity during time period [1,1.001] is given by;

vavg = [s(1.001) - s(1)]/(1.001 - 1)

= s(1.001) - s(1)

= [tex][5sin(1.001x) + 4cos(π1.001)] - [5sin(x) + 4cos(π)][/tex]

= [tex]5sin(1.001) - 5sin(1) + 4(cos(π1.001) - cos(π))[/tex]

= 5(sin(1.001) - sin(1)) cm/s≈ 0.0057 cm/s

(b) To estimate the instantaneous velocity of the particle when t = 1, we need to calculate the derivative of the displacement function s(t) with respect to time t.

The derivative of s(t) w.r.t t is given as follows;

s'(t) = 5xcos(xt) - 4πsin(πt)

At t = 1, the instantaneous velocity of the particle is given by;

[tex]s'(1) = 5xcos(x) - 4πsin(π)≈ 2.35x cm/s[/tex]

Therefore, the instantaneous velocity of the particle when t = 1 is approximately 2.35x cm/s.

To know more about derivative visit:

https://brainly.com/question/25324584

#SPJ11

Use synthetic division to find the quotient and the remainder when the first polynomial is divided by the second polynomial. 2x^(5)+2x^(4)-7x^(3)+x^(2)+x+2;x-2

Answers

The synthetic division can be used to find the quotient and the remainder when the first polynomial is divided by the second polynomial. The quotient is 2x^4 + 6x^3 + 5x^2 + 9x + 16 and the remainder is 7.

We are given the two polynomials:

2x^(5)+2x^(4)-7x^(3)+x^(2)+x+2

and x-2

We need to use synthetic division to find the quotient and remainder.

To perform the synthetic division, we should write the coefficients of the dividend in the first row

(the coefficients in order from highest degree to lowest degree).

Here, the highest degree is 5, so the first coefficient is 2.

The other coefficients are 2, -7, 1, 1, and 2.

Then we need to bring down the first coefficient, which is 2.  

The first number in the second row is 2 (the same as the first number in the previous row).

Then we multiply 2 by the divisor (-2) to get -4.

The sum of the two numbers 2 and -4 is -2.

We write this below -4. -2 is the second number of the second row.

Next, we multiply -2

(the second number of the second row) by -2 (the divisor) to get 4.

The sum of the two numbers -7 and 4 is -3. We write -3 below 4.

This is the third number of the second row. We can perform the same step as long as we need to get all the rows until we get the last remainder. 2, 2, -4, -2, -3, 7.

Therefore, the quotient is 2x^4 + 6x^3 + 5x^2 + 9x + 16 and the remainder is 7.

Answer:Thus, the synthetic division can be used to find the quotient and the remainder when the first polynomial is divided by the second polynomial. The quotient is 2x^4 + 6x^3 + 5x^2 + 9x + 16 and the remainder is 7.

To know more about quotient visit:

https://brainly.com/question/16134410

#SPJ11

Hong needs $5770 for a future project. He can invest $5000 now at an annual rate of 9.8%, compounded semiannually. Assuming that no
withdrawals are made, how long will it take for him to have enough money for his project?
Do not round any intermediate computations, and round your answer to the nearest hundredth.
m.

Answers

It will take approximately 3.30 years for Hong's investment to grow to $5770 at an annual interest rate of 9.8%, compounded semiannually.

To determine how long it will take for Hong to have enough money for his project, we need to calculate the time period it takes for his investment to grow to $5770.

The formula for compound interest is given by:

[tex]A = P(1 + r/n)^{(nt)[/tex]

Where:

A is the future value of the investment

P is the principal amount (initial investment)

r is the annual interest rate (in decimal form)

n is the number of times interest is compounded per year

t is the time period (in years)

In this case, Hong's initial investment is $5000, the annual interest rate is 9.8% (or 0.098 in decimal form), and the interest is compounded semiannually (n = 2).

We need to solve the formula for t:

[tex]5770 = 5000(1 + 0.098/2)^{(2t)[/tex]

Dividing both sides of the equation by 5000:

[tex]1.154 = (1 + 0.049)^{(2t)[/tex]

Taking the natural logarithm of both sides:

[tex]ln(1.154) = ln(1.049)^{(2t)[/tex]

Using the logarithmic identity [tex]ln(a^b) = b \times ln(a):[/tex]

[tex]ln(1.154) = 2t \times ln(1.049)[/tex]

Now we can solve for t by dividing both sides by [tex]2 \times ln(1.049):[/tex]

[tex]t = ln(1.154) / (2 \times ln(1.049)) \\[/tex]

Using a calculator, we find that t ≈ 3.30 years.

For similar question on annual interest rate.

https://brainly.com/question/29451175  

#SPJ8

The first term of a sequence is 19. The term-to-term
rule is to add 14 each time.
What is the nth term rule for the sequence?

Answers

Answer:

[tex]a_n=14n+5[/tex]

Step-by-step explanation:

[tex]a_n=a_1+(n-1)d\\a_n=19+(n-1)(14)\\a_n=19+14n-14\\a_n=14n+5[/tex]

Here, the common difference is [tex]d=14[/tex] since 14 is being added each subsequent term, and the first term is [tex]a_1=19[/tex].

length of the major axis of a horizotal ellipse with the center at (2,1) and coordinate of one of its vertices is (7,1)

Answers

The length of the major axis of the horizontal ellipse is 5 units.

The length of the major axis of a horizontal ellipse, we need to determine the distance between the center and one of its vertices.

Given that the center of the ellipse is at (2, 1) and one of its vertices is at (7, 1), we can calculate the distance between these two points.

The distance between two points (x₁, y₁) and (x₂, y₂) is given by the formula:

Distance = √((x₂ - x₁)² + (y₂ - y₁)²)

using this formula, we can find the distance between (2, 1) and (7, 1):

Distance = √((7 - 2)² + (1 - 1)²)

= √(5² + 0²)

= √25

= 5

Therefore, the length of the major axis of the horizontal ellipse is 5 units.

To know more about horizontal ellipse click here :

https://brainly.com/question/33295390

#SPJ4

For using a computerized financial news network for 50 min during prime time and 80 min during non-prime time, a customer was charged $12.50. A second customer was charged $14.55 for using the network
for 60 min of prime time and 90 min of non-prime time. Find the cost per minute for using the financial news network during prime time.

Answers

The cost per minute for using the financial news network during prime time is $0.13.

Let's consider that the cost per minute for using the financial news network during prime time is $x.

Using the given information, we can form the following equations:

For the first customer, the time used during prime time = 50 min, the time used during non-prime time = 80 min and the total cost = $12.50.

Hence, we can write the equation as:

50x + 80y = 12.50

For the second customer, the time used during prime time = 60 min, the time used during non-prime time = 90 min and the total cost = $14.55.

Hence, we can write the equation as:

60x + 90y = 14.55

We can use the elimination method to solve for x and y.

Multiplying the first equation by 9 and the second equation by -8, we get:

450x + 720y = 112.5    

-480x - 720y = -116.4  

150x - 120x + 240y - 180y = 37.50 - 29.10

30x + 60y = 8.40 (Equation 5)

Now we have a new equation (Equation 5) containing only the 'x' and 'y' terms. We can solve this equation for "x":

30x + 60y = 8.40

30x = 8.40 - 60y

x = (8.40 - 60y) / 30

x = 0.28 - 2y (equation 6)

Adding equation (1) and (2), we get:-30x = -3.9

Dividing by -30 on both sides, we get:x = 0.13

The financial news network during prime time is $0.13.

For more related questions on financial news:

https://brainly.com/question/31360936

#SPJ8

Find an equation of the plane through the three points given: P=(4,0,0),Q=(3,4,−4),R=(5,−1,−4)=−80

Answers

The equation of the plane is -16x - 12y - 4z + 64 = 0.

Given three points P = (4, 0, 0), Q = (3, 4, -4), R = (5, -1, -4) and a plane equation through the three points. We need to find the equation of the plane.

Let's start with the vector PQ and PR will lie on the plane

PQ vector = Q - P = (3, 4, -4) - (4, 0, 0)

                 = (-1, 4, -4)

PR vector = R - P = (5, -1, -4) - (4, 0, 0)

                = (1, -1, -4)

The normal vector of the plane will be perpendicular to both the above vectors.

N = PQ × PRN = (-1, 4, -4) x (1, -1, -4)

N = (-16, -12, -4)

The equation of the plane is of the form ax + by + cz = d. Now we will substitute any one of the three points to find the value of d. We use point P as P.

N + d = 0(-16)(4) + (-12)(0) + (-4)(0) + d = 0 +d = 64

The equation of the plane is -16x - 12y - 4z + 64 = 0. The plane is represented by the equation -16x - 12y - 4z + 64 = 0.

To know more about plane here:

https://brainly.com/question/27212023

#SPJ11

Assume that military aircraft use ejection seats designed for men weighing between 135.5 lb and 201lb. If women's weights are normally distributed with a mean of 160.1lb and a standard deviation of 49.5lb

what percentage of women have weights that are within thoselimits?

Are many women excluded with those specifications?

Answers

19.4% of women have weights that are within the limits of 135.5 lb and 201 lb and women's weights are normally distributed, we can assume that there are many women who fall outside these limits.

Mean can be defined as the average of all the values in a dataset. Standard deviation can be defined as a measure of the spread of a dataset. Percentage is a way of representing a number as a fraction of 100.

Assume that military aircraft use ejection seats designed for men weighing between 135.5 lb and 201 lb.

If women's weights are normally distributed with a mean of 160.1 lb and a standard deviation of 49.5 lb, we need to find out what percentage of women have weights that are within those limits.

To solve this, we need to standardize the weights using the formula z = (x - μ) / σ, where x is the weight of a woman, μ is the mean weight of women and σ is the standard deviation of women's weight.

We can then use a standard normal distribution table to find the percentage of women who fall between the two given limits:

z for the lower limit = (135.5 - 160.1) / 49.5 = -0.498z for the upper limit = (201 - 160.1) / 49.5 = 0.826

The percentage of women with weights between these limits is given by the area under the standard normal curve between -0.498 and 0.826.

From a standard normal distribution table, we can find this area to be 19.4%.

Therefore, 19.4% of women have weights that are within the limits of 135.5 lb and 201 lb.

Since women's weights are normally distributed, we can assume that there are many women who fall outside these limits.

To know more about Percentage visit:

https://brainly.com/question/30931854

#SPJ11

A ladder 13 feet long is leaning against a vertical wall. The top of the ladder is sliding down the wall at a rate of 2 feet per second. (a) Draw and label the diagram for this application problem. How fast is the foot of the ladder moving away from the wall when the foot is 5 feet from the base of the wall? (b) Find the rate at which the angle between the ladder and the wall is changing when the foot of the ladder is 5 feet from the base of the wall.

Answers

The rate at which the angle between the ladder and the wall is changing when the foot of the ladder is 5 feet from the base of the wall is approximately 42.32°/s.

(b)Let θ be the angle between the ladder and the wall.

Then, sin θ = BC/AB or BC = AB sin θ

Since AB = 13 ft, we have BC = 13 sin θ

Differentiating both sides of the equation with respect to time t,

we get:

d/dt (BC) = d/dt (13 sin θ)13 (cos θ) (dθ/dt)

= 13 (cos θ) (dθ/dt)

= 13 (d/dt sin θ)13 (dθ/dt)

= 13 (cos θ) (d/dt sin θ)

Using the fact that sin θ = BC/AB, we can express the equation as:

dθ/dt = (AB/BC) (d/dt BC)

We know that AB = 13 ft and dBC/dt = 4.8 ft/s when BC = 5 ft.

Therefore,θ = sin⁻¹(BC/AB)

= sin⁻¹(5/13)θ ≈ 23.64°

Now, dθ/dt = (13/5) (4.8/13)

= 0.7392 rad/s

≈ 42.32°/s

Therefore, the rate at which the angle between the ladder and the wall is changing when the foot of the ladder is 5 feet from the base of the wall is approximately 42.32°/s.

Know more about angle  here:

https://brainly.com/question/25770607

#SPJ11

1. Find the area of the region that is between the curves y=x^{2} and y=x+2 2. Find the area of the region bounded by the curves y=sin x, y=cos x,{x}=0 , and x=\frac{π}

Answers

The region that is between the curves y = x² and y = x + 2 is shown in the figure. Hence, the area of the region that is between the curves y = x² and y = x + 2 is given by Area = ∫ a b (x + 2 - x²) dx.

The intersection points of the curves y = x² and y = x + 2 are given by:

x² = x + 2

=> x² - x - 2 = 0

=> (x - 2) (x + 1) = 0.

The intersection points of the curves y = x² and y = x + 2 are given by:

x = 2, and x = -1.

Therefore, the required area is given by:

∫ ₂ -₁ [(x + 2) - x²] dx

= ∫ ₂ -₁ (2 - x - x²) dx

= [2x - (x²/2) - (x³/3)] from 2 to -1

= [(-8/3) + (4/2) + 4] - [(4 - 2 + 0)]/2

= [8/3 + 4] - [2]/2= 20/3 square units

The area of the region bounded by the curves y = sin x, y = cos x, x = 0, and x = π/4 is shown in the figure below.

The required area is given by:

∫ 0 π/4 (cos x - sin x) dx

= [sin x + cos x] from 0 to π/4

= [sin (π/4) + cos (π/4)] - [sin 0 + cos 0]

= [(√2/2) + (√2/2)] - [0 + 1]

= √2 - 1 square units.

To know more about intersection points visit:

https://brainly.com/question/14217061

#SPJ11

write the equation of line with slope ( 3)/(4) and y-intercept (0,-8) and find two move ponts on line solve

Answers

In summary, the equation of the line is y = (3/4)x - 8, and two additional points on the line are (4, -5) and (-2, -19/2).

The equation of a line can be expressed in slope-intercept form as:

y = mx + b

where:

m represents the slope of the line, and

b represents the y-intercept.

Given that the slope (m) is 3/4 and the y-intercept (0, -8), we can substitute these values into the equation:

y = (3/4)x - 8

To find two additional points on the line, we can select any x-values and substitute them into the equation to calculate the corresponding y-values.

Let's choose x = 4:

y = (3/4)(4) - 8

y = 3 - 8

y = -5

Therefore, the point (4, -5) lies on the line.

Now, let's choose x = -2:

y = (3/4)(-2) - 8

y = -3/2 - 8

y = -19/2

Hence, the point (-2, -19/2) is also on the line.

To know more about equation,

https://brainly.com/question/21378402

#SPJ11

23. Is it an SRS? A corporation employs 2000 male and 500 female engineers. A stratified random sumple of 200 male and 50 female engineers gives each engineer I chance in 10 to be chosen. This sample design gives every individual in the population the same chance to be chosen for the sample. Is it an SRS? Explain your answer. 25. High-speed Internet laying fiber-optic cable is expensive. Cable companics want to make sure that if they extend their lines out to less dense suburban or rural areas, there will be sufficient demand and the work will be costeffective. They decide to conduct a survey to deterumine the proportion of homsehokds in a rural subdivision that would buy the service. They select a simple tandom sample of 5 blocks in the subdivision and survey each family that lives on one of those blocks. (a) What is the name for this kind of sampling method? (b) Give a possible reason why the cable company chose this method.

Answers

23. A stratified random sample design was used instead of a simple random sample in the given scenario. It is not an SRS. This is because a simple random sample provides each individual in the population with an equal chance of being chosen for the sample.

But, in this case, different subgroups (males and females) of the population were divided before sampling. Instead of drawing samples randomly from the entire population, the sample was drawn separately from each stratum in a stratified random sample design. The sizes of these strata are proportional to their sizes in the population.

Therefore, a stratified random sample is not the same as a simple random sample.25.

(a) The sampling method used by the cable company is called Cluster Sampling.

b) Cable companies use cluster sampling method when the population being sampled is geographically large and scattered over a wide area. In such cases, surveying each member of the population can be difficult, time-consuming, and expensive. The companies divide the population into clusters, which are geographic groupings of the population. They then randomly select some of these clusters for inclusion in the survey. Finally, they collect data on all members of each selected cluster.

This method was chosen by the cable company because it is easier to contact respondents within the selected clusters and less costly than a simple random sample.

to know more about SRS

https://brainly.com/question/33625564

#SPJ11

Branches**: Complex cost structure An airline describes airfare as follows. A normal ticket's base cost is $300. Persons aged 60 or over have a base cost of $290. Children 2 or under have $0 base cost. A carry-on bag costs $10. A first checked bag is free, second is $25, and each additional is $50. Given inputs of age, carry-on ( 0 or 1 ), and checked bags ( 0 or greater), compute the total airfare. Hints: - First use an if-else statements to assign airFare with the base cost - Use another if statement to update airFare for a carryOn - Finally, use another if-else statement to update airFare for checked bags - Think carefully about what expression correctly calculates checked bag cost when bags are 3 or more 4007822448304.9×329y7 \begin{tabular}{|l|l} LAB & 3.17.1: PRACTICE: Branches**: Complex cost structure \\ ACTIITY & \end{tabular} main.java Load default template... 1 import java.util. Scanner; 3 public class main \{ 4 public static void main(String □ args) \{ 5 Scanner scnr = new Scanner(System. in); 6 int passengerAge; 7 int carryons; 8 int checkedBags; 9 int airFare; 11 passengerAge = scnr, nextInt () 12 carryOns = scnr, nextInt(); 13 checkedBags = scnr. nextInt (; 14 / / * Type your code here. */

Answers

We use another if-else statement to update airFare for checked bags, taking into account the correct expression for calculating the checked bag cost when there are 3 or more bags.

import java.util.Scanner;

public class Main {

   public static void main(String[] args) {

       Scanner scnr = new Scanner(System.in);

       int passengerAge;

       int carryOns;

       int checkedBags;

       int airFare;

       passengerAge = scnr.nextInt();

       carryOns = scnr.nextInt();

       checkedBags = scnr.nextInt();

       // Calculate base cost based on passenger's age

       if (passengerAge >= 60) {

           airFare = 290;

       } else if (passengerAge <= 2) {

           airFare = 0;

       } else {

           airFare = 300;

       }

       // Add cost for carry-on bag

       if (carryOns == 1) {

           airFare += 10;

       }

       // Add cost for checked bags

       if (checkedBags == 1) {

           airFare += 25;

       } else if (checkedBags >= 2) {

           airFare += 25 + 50 * (checkedBags - 1);

       }

       System.out.println("Total Airfare: $" + airFare);

   }

}

In this code, we first use if-else statements to assign the base cost (airFare) based on the passenger's age. Then, we use another if statement to update airFare for the carry-on bag. Finally, we use another if-else statement to update airFare for checked bags, taking into account the correct expression for calculating the checked bag cost when there are 3 or more bags.

To know more about expression, visit:

https://brainly.com/question/28170201

#SPJ11

The population of a country dropped from 52.4 million in 1995 to 44.6 million in 2009. Assume that P(t), the population, in millions, 1 years after 1995, is decreasing according to the exponential decay
model
a) Find the value of k, and write the equation.
b) Estimate the population of the country in 2019.
e) After how many years wil the population of the country be 1 million, according to this model?

Answers

Assume that P(t), the population, in millions, 1 years after 1995, is decreasing according to the exponential decay model. A) The value of k = e^(14k). B) Tthe population of the country in 2019 = 33.6 million. E) After about 116 years (since 1995), the population of the country will be 1 million according to this model.

a) We need to find the value of k, and write the equation.

Given that the population of a country dropped from 52.4 million in 1995 to 44.6 million in 2009.

Assume that P(t), the population, in millions, 1 years after 1995, is decreasing according to the exponential decay model.

To find k, we use the formula:

P(t) = P₀e^kt

Where: P₀

= 52.4 (Population in 1995)P(t)

= 44.6 (Population in 2009, 14 years later)

Putting these values in the formula:

P₀ = 52.4P(t)

= 44.6t

= 14P(t)

= P₀e^kt44.6

= 52.4e^(k * 14)44.6/52.4

= e^(14k)0.8506

= e^(14k)

Taking natural logarithm on both sides:

ln(0.8506) = ln(e^(14k))

ln(0.8506) = 14k * ln(e)

ln(e) = 1 (since logarithmic and exponential functions are inverse functions)

So, 14k = ln(0.8506)k = (ln(0.8506))/14k ≈ -0.02413

The equation for P(t) is given by:

P(t) = P₀e^kt

P(t) = 52.4e^(-0.02413t)

b) We need to estimate the population of the country in 2019.

1 year after 2009, i.e., in 2010,

t = 15.P(15)

= 52.4e^(-0.02413 * 15)P(15)

≈ 41.7 million

In 2019,

t = 24.P(24)

= 52.4e^(-0.02413 * 24)P(24)

≈ 33.6 million

So, the estimated population of the country in 2019 is 33.6 million.

e) We need to find after how many years will the population of the country be 1 million, according to this model.

P(t) = 1P₀ = 52.4

Putting these values in the formula:

P(t) = P₀e^kt1

= 52.4e^(-0.02413t)1/52.4

= e^(-0.02413t)

Taking natural logarithm on both sides:

ln(1/52.4) = ln(e^(-0.02413t))

ln(1/52.4) = -0.02413t * ln(e)

ln(e) = 1 (since logarithmic and exponential functions are inverse functions)

So, -0.02413t

= ln(1/52.4)t

= -(ln(1/52.4))/(-0.02413)t

≈ 115.73

Therefore, after about 116 years (since 1995), the population of the country will be 1 million according to this model.

To know more about exponential visit:

https://brainly.com/question/29160729

#SPJ11

I neew help with e,f,g
(e) \( \left(y+y x^{2}+2+2 x^{2}\right) d y=d x \) (f) \( y^{\prime} /\left(1+x^{2}\right)=x / y \) and \( y=3 \) when \( x=1 \) (g) \( y^{\prime}=x^{2} y^{2} \) and the curve passes through \( (-1,2)

Answers

There is 1st order non-linear differential equation in all the points mentioned below.

(e) \(\left(y+yx^{2}+2+2x^{2}\right)dy=dx\)

This is a first-order nonlinear ordinary differential equation. It is not linear, autonomous, or homogeneous.

(f) \(y^{\prime}/\left(1+x^{2}\right)=x/y\) and \(y=3\) when \(x=1\)

This is a first-order nonlinear ordinary differential equation. It is not linear, autonomous, or homogeneous. The initial condition \(y=3\) when \(x=1\) provides a specific point on the solution curve.

(g) \(y^{\prime}=x^{2}y^{2}\) and the curve passes through \((-1,2)\)

This is a first-order nonlinear ordinary differential equation. It is not linear, autonomous, or homogeneous. The given point \((-1,2)\) is an initial condition that the solution curve passes through.

There is 1st order non-linear differential equation in all the points mentioned below.

Learn more differential equation here:

https://brainly.com/question/32645495

#SPJ11

02:12:34 Calculate the GPA of a student with the following grades: B (11 hours ), A (18 hours ), F (17 hours ), Note that an A is equivalent to 4.0, a B is equivalent to a 3.0, a C is equivalent to a

Answers

The GPA of the student is 2.28.

To calculate the GPA of a student with the following grades: B (11 hours), A (18 hours), F (17 hours), we can use the following steps:Step 1: Find the quality points for each gradeThe quality points for each grade can be found by multiplying the equivalent grade points by the number of credit hours:B (11 hours) = 3.0 x 11 = 33A (18 hours) = 4.0 x 18 = 72F (17 hours) = 0.0 x 17 = 0Step 2: Find the total quality pointsThe total quality points can be found by adding up the quality points for each grade:33 + 72 + 0 = 105Step 3: Find the total credit hoursThe total credit hours can be found by adding up the credit hours for each grade:11 + 18 + 17 = 46Step 4: Calculate the GPAThe GPA can be calculated by dividing the total quality points by the total credit hours:GPA = Total quality points / Total credit hoursGPA = 105 / 46GPA = 2.28Therefore, the GPA of the student is 2.28.

Learn more about point :

https://brainly.com/question/28224145

#SPJ11

Use the following problem to answer questions 7 and 8. MaxC=2x+10y 5x+2y≤40 x+2y≤20 y≥3,x≥0 7. Give the corners of the feasible set. a. (0,3),(0,10),(6.8,3),(5,7.5) b. (0,20),(5,7.5),(14,3) c. (5,7.5),(6.8,3),(14,3) d. (0,20),(5,7.5),(14,3),(20,0) e. (0,20),(5,7.5),(20,0) 8. Give the optimal solution. a. 200 b. 100 c. 85 d. 58 e. 40

Answers

The corners of the feasible set are:

b. (0,20), (5,7.5), (14,3)

To find the corners of the feasible set, we need to solve the given set of inequalities simultaneously. The feasible set is the region where all the inequalities are satisfied.

The inequalities given are:

5x + 2y ≤ 40

x + 2y ≤ 20

y ≥ 3

x ≥ 0

From the inequality x + 2y ≤ 20, we can rearrange it to y ≤ (20 - x)/2.

Since y ≥ 3, we can combine these two inequalities to get 3 ≤ y ≤ (20 - x)/2.

From the inequality 5x + 2y ≤ 40, we can rearrange it to y ≤ (40 - 5x)/2.

Since y ≥ 3, we can combine these two inequalities to get 3 ≤ y ≤ (40 - 5x)/2.

Now, let's check the corners by substituting the values:

For (0, 20):

3 ≤ 20/2 and 3 ≤ (40 - 5(0))/2, which are both true.

For (5, 7.5):

3 ≤ 7.5 ≤ (40 - 5(5))/2, which are all true.

For (14, 3):

3 ≤ 3 ≤ (40 - 5(14))/2, which are all true.

Therefore, the corners of the feasible set are (0,20), (5,7.5), and (14,3).

The corners of the feasible set are (0,20), (5,7.5), and (14,3) - option d.

The optimal solution is:

c. 85

To find the optimal solution, we need to evaluate the objective function at each corner of the feasible set and choose the maximum value.

The objective function is MaxC = 2x + 10y.

For (0,20):

MaxC = 2(0) + 10(20) = 0 + 200 = 200.

For (5,7.5):

MaxC = 2(5) + 10(7.5) = 10 + 75 = 85.

For (14,3):

MaxC = 2(14) + 10(3) = 28 + 30 = 58.

Therefore, the maximum value of the objective function is 85, which occurs at the corner (5,7.5).

The optimal solution is 85 - option c.

To know more about corners, visit;
https://brainly.com/question/30466188
#SPJ11

A bacteria culture is started with 250 bacteria. After 4 hours, the population has grown to 724 bacteria. If the population grows exponentially according to the foula P_(t)=P_(0)(1+r)^(t) (a) Find the growth rate. Round your answer to the nearest tenth of a percent.

Answers

The growth rate is 19.2% (rounded to the nearest tenth of a percent).

To find the growth rate, we can use the formula P_(t)=P_(0)(1+r)^(t), where P_(0) is the initial population, P_(t) is the population after time t, and r is the growth rate.

We know that the initial population is 250 and the population after 4 hours is 724. Substituting these values into the formula, we get:

724 = 250(1+r)^(4)

Dividing both sides by 250, we get:

2.896 = (1+r)^(4)

Taking the fourth root of both sides, we get:

1.192 = 1+r

Subtracting 1 from both sides, we get:

r = 0.192 or 19.2%

Therefore, the value obtained is 19.2% which is the growth rate.

To know more about growth rate refer here:

https://brainly.com/question/18485107#

#SPJ11

Your work colleague has estimated a regression to predict the monthly return of a mutual fund (Y) based on the return of the S&P 500 (X). Your colleague expected that the "true" relationship is Y = 0.01 + (0.84)(X). The regression was estimated using 100 observations of prior monthly returns in excel and the following results for the variable X were shown in the excel output: Coefficient: 1.14325 Standard error: 0.33138 t Stat: 3.44997 Should the hypothesis that the actual, true slope coefficient (i.e., the coefficient for X) is as your colleague expected to be rejected at the 1% level? You decided to calculate a t-stat/z-score to test this, which you will then compare to the critical value of 2.58. What is the t-stat/z-score for performing this test? Question 4 in the practice problems maybe be helpful. Express your answer rounded and accurate to the nearest 2 decimal places.

Answers

The t-stat/z-score is 0.92. To calculate the t-statistic/z-score, we need to use the formula:

t-stat/z-score = (estimated slope - hypothesized slope) / standard error of estimated slope

where the estimated slope is 1.14325, the hypothesized slope is 0.84, and the standard error of estimated slope is 0.33138.

So,

t-stat/z-score = (1.14325 - 0.84) / 0.33138

= 0.30387 / 0.33138

= 0.9175

Rounding to the nearest two decimal places, the t-stat/z-score is 0.92.

Since the absolute value of the t-statistic/z-score is less than the critical value of 2.58 at the 1% significance level, we fail to reject the hypothesis that the actual, true slope coefficient is as expected by your colleague.

learn more about t-statistic here

https://brainly.com/question/30765535

#SPJ11

For each system, determine whether it has a unique solution (in this case, find the solution), infinitely many solutions, or no solutions. a.
-2x+2y= 10
-4x+4y = 20
A. Infinitely many solutions
B. Unique solution: -0, -0
C. No solutions
D. Unique solution: z=-5, y=0
E. Unique solution: -
F. None of the above

Answers

The solution to the system of equations is \boxed{\textbf{(D) } \text{Unique solution: }x=-5, y=0}.

Let us solve the following system of equations: \begin{aligned}-2x+2y &= 10\\-4x+4y &= 20\end{aligned}$$

We can simplify the second equation by dividing both sides by 4.

This will give us the same equation as the first. \begin{aligned}-2x+2y &= 10\\-x+y &= 5\end{aligned}

This system of equations can be solved by adding the equations together.

-2x + 2y + (-x + y) = 10 + 5-3x + 3y = 15 -3(x - y) = 15 x - y = -5

Therefore, the system of equations has a unique solution. The solution is \begin{aligned}x - y &= -5\\x &= -5 + y\end{aligned}

Therefore, we can use either equation in the original system of equations to solve for y-2x+2y= 10-2(-5 + y) + 2y = 10, 10 - 2y + 2y = 10, 0 = 0

Since 0 = 0, the value of y does not matter. We can choose any value for y and solve for x. For example, if we let y = 0, then x - y = -5x - 0 = -5 x = -5

Therefore, the solution to the system of equations is \boxed{\textbf{(D) } \text{Unique solution: }x=-5, y=0}.

To know more about system of equations visit:
brainly.com/question/33289088

#SPJ11

A machine that manufactures automobile parts produces defective parts 15% of the time. If 10 parts produced by this machine are randomly selected, what is the probability that fewer than 2 of the parts are defective? Carry your intermediate computations to at least four decimal places, and round your answer to two decimal places.

Answers

The answer is 0.00.

Given information:

Probability of success, p = 0.85 (producing a non-defective part)

Probability of failure, q = 0.15 (producing a defective part)

Total number of trials, n = 10

We need to find the probability of getting fewer than 2 defective parts, which can be calculated using the binomial distribution formula:

P(X < 2) = P(X = 0) + P(X = 1)

Using the binomial distribution formula, we find:

P(X = 0) = (nCx) * (p^x) * (q^(n - x))

        = (10C0) * (0.85^0) * (0.15^10)

        = 0.00000005787

P(X = 1) = (nCx) * (p^x) * (q^(n - x))

        = (10C1) * (0.85^1) * (0.15^9)

        = 0.00000254320

P(X < 2) = P(X = 0) + P(X = 1)

        = 0.00000005787 + 0.00000254320

        = 0.00000260107

        = 0.0003

Rounding the answer to two decimal places, the probability that fewer than 2 of the parts are defective is 0.00.

Learn more about Probability

https://brainly.com/question/31828911

#SPJ11

Suppose that the quadratic equation S=0.0654x^(2)-0.801x+9.64 models sales of new cars, where S represents sales in millions, and x=0 represents 2000,x=1 represents 2001, and so on. Which equation sho

Answers

The equation that should be used to determine sales in 2010 is S = 8.17 million.

To determine sales in 2010, we need to find the value of x that corresponds to that year.

Since x=0 represents 2000 and x increases by 1 for each subsequent year, we can calculate the value of x for 2010 by subtracting 2000 from the year.

2010 - 2000 = 10

Therefore, x = 10 represents the year 2010 in this context.

To determine the sales in 2010, we substitute x=10 into the quadratic equation [tex]S = 0.0654x^2 - 0.801x + 9.64:[/tex]

[tex]S = 0.0654(10)^2 - 0.801(10) + 9.64[/tex]

= 0.0654(100) - 0.801(10) + 9.64

= 6.54 - 8.01 + 9.64

= 8.17.

Hence, the equation that should be used to determine sales in 2010 is S = 8.17 million.

Note: The calculation assumes that the quadratic equation accurately models the sales of new cars over the given time period and that there are no other factors affecting sales.

For similar question on sales.

https://brainly.com/question/25743891  

#SPJ8

Question: Suppose that the quadratic equation S=0.0654x^(2)-0.801x+9.64 models sales of new cars, where S represents sales in millions, and x=0 represents 2000,x=1 represents 2001, and so on. Which equation should be used to determine sales in 2010?

Factor the polynomial x ^2+5x−14. Your answer can be written as (x+A)(x+B) where A

Answers

To factorize the polynomial x² + 5x - 14, the factors of -14 must be determined. They are: -1 and 14, 1 and -14, -2 and 7, and 2 and -7.

However, it is observed that the product of 7 and -2 is -14, and the sum of the two factors is 5.

This suggests that -2 and 7 should be the factors of the polynomial x² + 5x - 14.

Thus, (x - 2)(x + 7) can be written as the factorization of the given polynomial.

This can be shown by expanding the product: (x - 2)(x + 7) = x² + 7x - 2x - 14 = x² + 5x - 14

Therefore, the factorization of the polynomial x² + 5x - 14 is (x - 2)(x + 7).

For more such questions on polynomial, click on:

https://brainly.com/question/1496352

#SPJ8

A side of the triangle below has been
extended to form an exterior angle of 64°.
Find the value of x.
64°
to
49°

Answers

Answer: 116

Step-by-step explanation:

supplementary angle = 180 degrees

180-64 = x

x=116

Answer:     X  =   116 degrees

                  Therefore, The value of X:    

                  X  =   116 degrees  

Step-by-step explanation:

SOLVE:  

SUM of ANGLES of  TRIANGLES EQUALS TO  180 Degrees

Exterior Angle   =   Sum of Opposite Interior Angle

64 degrees        =     49 degrees    +    y

                  y       =     15 degrees

X   +  y  +  49 degrees    =    180 degrees

X   +  15  +  49 degrees   =   180 degrees

                                  X    =   180 degrees  -   64 degrees

                                   X   =   116 degrees

DRAW A CONCLUSION:

    Therefore, The value of X:    

                  X  =   116 degrees  

I hope this helps you!

Hi, please help me with this question. I would like an explanation of how its done, the formula that is used, etc.
The largest of 123 consecutive integers is 307. What is the smallest?

Answers

Therefore, the smallest of the 123 consecutive integers is 185.

To find the smallest of 123 consecutive integers when the largest is given, we can use the formula:

Smallest = Largest - (Number of Integers - 1)

In this case, the largest integer is 307, and we have 123 consecutive integers. Plugging these values into the formula, we get:

Smallest = 307 - (123 - 1)

= 307 - 122

= 185

To know more about integers,

https://brainly.com/question/15015575

#SPJ11

We described implicit differentiation using a function of two variables. This approach applies to functions of three or more variables. For example, let's take F(x, y, z) = 0 and assume that in the part of the function's domain we are interested in,∂F/∂y ≡F′y ≠ 0. Then for y = y(x, z) defined implicitly via F(x, y, z) = 0, ∂y(x,z)/∂x ≡y′x (x,z)= −F′x/F′y. Now, assuming that all the necessary partial derivatives are not zeros, find x′y. y′z.z′x .

Answers

The value of  x′y = -∂F/∂y / ∂F/∂x , y = y(x, z): y′z = -∂F/∂z / ∂F/∂y and z′x = -∂F/∂x / ∂F/∂z. The expression x′y represents the partial derivative of x with respect to y.

Using the implicit differentiation formula, we can calculate x′y as follows: x′y = -∂F/∂y / ∂F/∂x.

Similarly, y′z represents the partial derivative of y with respect to z. To find y′z, we use the implicit differentiation formula for y = y(x, z): y′z = -∂F/∂z / ∂F/∂y.

Lastly, z′x represents the partial derivative of z with respect to x. Using the implicit differentiation formula, we have z′x = -∂F/∂x / ∂F/∂z.

These expressions allow us to calculate the derivatives of the variables x, y, and z with respect to each other, given the implicit function F(x, y, z) = 0. By taking the appropriate partial derivatives and applying the division formula, we can determine the values of x′y, y′z, and z′x.

Learn more about partial derivative here :  brainly.com/question/32387059

#SPJ11

\( A=\left[\begin{array}{cc}-1 & 1 / 2 \\ 0 & 1\end{array}\right] \)

Answers

The matrix \( A \) is a 2x2 matrix with the elements -1, 1/2, 0, and 1. It represents a linear transformation that scales the y-axis by a factor of 1 and flips the x-axis.

The given matrix \( A \) represents a linear transformation in a two-dimensional space. The first row of the matrix corresponds to the coefficients of the transformation applied to the x-axis, while the second row corresponds to the y-axis. In this case, the transformation is defined as follows:

1. The first element of the matrix, -1, indicates that the x-coordinate will be flipped or reflected across the y-axis.

2. The second element, 1/2, represents a scaling factor applied to the y-coordinate. It means that the y-values will be halved or compressed.

3. The third element, 0, implies that the x-coordinate will remain unchanged.

4. The fourth element, 1, indicates that the y-coordinate will be unaffected.

Overall, the matrix \( A \) performs a transformation that reflects points across the y-axis while maintaining the same x-values and compressing the y-values by a factor of 1/2.

For more information on matrix visit: brainly.com/question/33536359

#SPJ11

The director of training for an electronic equipment manufacturer is interested in determining whether different training methods have an effect on the productivity of the assembly-line employees. She randomly assigns 42 recently hired employees into two groups of 21. The first group receives a computer-assisted, Individual-based training program and the other receives a team-based training program. Upon completion of the training, the employees are evaluated on the time (in seconds) it takes to assemble a part. The results are in the data file training.

a) Assuming that the variances in the populations of training methods are equal, is there evidence of a difference between the mean assembly times (in seconds) of employees trained in a computer-assisted, individual-based program and those trained in a team-based program? (Use a 0.05 level of significance.)

b) What other assumptions are necessary in (a)?

Answers

To determine if there is evidence of a difference between the mean assembly times of employees trained in a computer-assisted, individual-based program and those trained in a team-based program, we can perform a two-sample t-test assuming equal variances.

a) Assumptions for the two-sample t-test:

1. Random sampling: The employees were randomly assigned to the two training groups. This assumption is satisfied as per the given information.

2. Independent samples: The assembly times of employees trained in the computer-assisted, individual-based program are independent of the assembly times of employees trained in the team-based program. This assumption is satisfied based on the random assignment of employees to the groups.

3. Normality: The assembly times within each group should follow a normal distribution. This assumption should be checked separately for each group using statistical tests or graphical methods such as normal probability plots or histograms.

4. Equal variances: The variances of assembly times in the two groups should be equal. This assumption can be tested using statistical tests such as Levene's test or by examining the ratio of the sample variances.

b) Other necessary assumptions:

1. Homogeneity of variances: As stated in the problem, the assumption is that the variances in the populations of the two training methods are equal. This assumption can be tested using statistical tests as mentioned above.

2. Independence of observations: The assembly times of one employee should not be influenced by the assembly times of other employees. This assumption is satisfied based on the information provided.

Once these assumptions are met, we can proceed with the two-sample t-test to test for a difference in the mean assembly times between the two training methods.

The test will provide a p-value that can be compared to the chosen level of significance (0.05) to determine if there is sufficient evidence to reject the null hypothesis of equal means.

Learn more about Sampling here:

https://brainly.com/question/31890671

#SPJ4

Other Questions
there is an excess supply of the good. b) quantity supplied exceeds quantity demanded. c) the current price is below its equilibrium price. d) all of the above are correct. In the context of the managerial skills development course ; Conficts of interest and cutural and personal differences may lead toMisunderstandingValue conflictsEthical DilemmasGreed and team frictions. Find the equation for the plane through the points Po(-5,-4,-3), Qo(4,4,4), and Ro(0, -5,-3).Using a coefficient of 1 for x, the equation of the plane is Develop a JavaScript using a function named "distance" that calculates the distance between two points on a graph, (x1, y1) and (x2, y2). Create an HTML form for the user to input the four (4) numbers. You will need to use the Pythagorean Theorem in creating your calculation Multiple users share a 10Mbps link. Each user requires 10Mbps when transmitting, but each user transmits for only 10% of the time. When circuit switching is used, how many users can be supported? Write an essay on the following topic. Is homework is imp? Use the quadratic formula to solve the equation. Use a calculator to give solutions correct to the nearest tenth. 3x^(2)=2+8x Albert was a partner in a management consulting firm that specialized in assisting small businesses in the area of financial planning. The firm did not perform income tax services. However, since he had personal expertise in tax accounting, he frequently assisted friends, as well as some firm clients, in preparing their income tax returns. Usually he borrowed the firm's adding machine and other office supplies to perform this work from his home unbeknownst to his partners. He may be required toa.All of the responses are possibleb.account to the other partners for any money received from his income tax service.c.retire from the partnership for breach of warranty of authority.d.None of the responses are possible.e.assign his share in the partnership to another to take his place in the partnership. The present value equals the future value when ... Select one: a. interest rate is lower than zero b. none of the above c. interest rate is zero d. interest rate is higher than zero if a system's entire set of microoperations consists of 41 statements, how many bits must be used for its microop code? HW Score: 87.5%,14 of 16 points at which the ball reaches its maximum height and find the maximum height View Policies Current Attempt in Progress For 2022, Pina Colada Corp.reported net income of $36000, net sales $351500, and average share outstanding of 15000 No preferred dividends were paid. Earnings per share is $2.40 O $25.15 $0.59 50.24 You have to design your own Java console application about any valid problem that your application must solve. Your solution can include solving a business problem, a new idea or even a game. Your application must make use of concepts such as arrays, loops, inheritance, constructors and information hiding. Output must be shown in the form of a report using the console. In your solution, make use of as many concepts, and components dealt with in this course, but pay special attention to the following learning units: - Learning Unit 1: Advanced arrays. - Learning Unit 2: Introduction to inheritance. Which of the following is a measured value? A. 20 desks B. 9 kilograms C. 4.67 centimeters D. 1 yard =3 feet a. A only b. Conly c. A&D d. B&C e. B,C&D True or False. Goals are broad general statements of what is expected to be accomplished. Programming C Help: How can I make two inputs in the same line work?Hellp, I was wondering how I can add two inputs separated by a space work?This is a small program I wrote to try understand the concept:-----------------#include int main(){//seperate scanf statementschar letter; int number;printf("Enter a letter and then a number:");scanf("%c", letter);scanf("%d", number);printf("Your letter: %c", letter);printf("\nYour number: %d", number);return 0;}-----------------The input I want to write is "b 7" so the output should be:Your letter: bYour number: 7Thank you! Make a stem-and-leaf display for these data. Use an interval width of 5 . 120,122,123,123,124,125,126,125,126,130,134,135,135,138,147 Consider a neural network with 5 input features x1 to x5 and the output of the Neural Network has values Z1=2.33, Z2= -1.46, Z3=0.56.The Target output of the function is [1,0,1] Calculate the probabilities using Soft Max Function and estimate the loss using cross-entropy. the goods-producing industries have a higher rate of nonfatal injuries and illnesses than the service-providing industries. TRUE or FALSE Show that the class of context free languages is closed under the union operation (construction and proof). The construction should be quite simple. To help you get started: G U=(V 1V 2{S},,R 1R 2{SS 1S 2},S), where G 1=(V 1,,R 1, S 1) and G 2=(V 2,,R 2, S 2) are CFGs. We assume that the rules and variables of G 1and G 2are disjoint. You still need to show that L(G U)=L(G 1)U(G 2).